5
$\begingroup$

I do understand the math part of the transformation, but I have a problem understanding why exactly there will be a "triangle" (to say it simply). Pretty much all the textbooks and videos describe this in the same way, and if you are not sure about what I mean exactly, please watch this video https://youtu.be/feBT0Anpg4A?si=CRqsWs16LC_G8Gfj&t=736 (skip at 12:16)

Let's say we are standing on the ground, and there is a man with a laser standing on a moving platform (with fairly large speed) with a mirror at the top. The man goes from -x to +x. We are standing exactly at the x=0, y=0 point. Once the man's x-y coordinates become the same (get in front of us basically), he shoots up a light beam in the y direction.

Every explanation would say that the light beam would reach the mirror and bounce back. In that process, it would take a trajectory similar to a triangle, and based on that then we can derive the formula for the transformation.

Now, this is possible because the light beam sort of moves with the platform. Just like a ball that is thrown upwards (no air resistance) on a moving boat does not fall backwards (to the opposite side of the direction of the movement) and lands back on the deck, the light beam too "lands" at the very same point it was shot from. My question is, why?

Why would the light beam go upwards and also "follow" the platform? For example, when the man shoots a light beam upwards at x=0, why wouldn't the light beam follow the y-axis? So that when man's position is +x1 after some delta t, the beam's x position is still zero since it just keeps going in the same direction.

When the platform comes to x=0, let's say I also shoot my white beam upwards. My light beam will keep moving across the y-axis. Why wouldn't his light beam also move parallel to my light beam? Why does it have to follow the movement of the platform? Its y position is equal to the y position of my beam, but its x position is not equal to the x position of my light beam. It is equal to the x position of the center of the platform (Because the launcher is in the middle of the platform).

If it were a ball being thrown upwards, it would make sense since that ball already has the momentum of the boa, so it will cover a track that looks like a triangle from our perspective. Why would the light beam also do that?

New contributor
Eduard is a new contributor to this site. Take care in asking for clarification, commenting, and answering. Check out our Code of Conduct.
$\endgroup$
2
  • $\begingroup$ From your point of view (frame of reference), your laser has no horizontal (x) speed, but the one shot from the moving platform does. But from the man's perspective it's the oppposite. Neither is entitled with "being the right one". But... what's the mechanism? How is the inertia in the laser pointer transferred to the photon being produced right then and there?? $\endgroup$ Commented 2 days ago
  • $\begingroup$ You may also enjoy What would your reflection look like if a mirror was going close to the speed of light? $\endgroup$ Commented yesterday

6 Answers 6

6
$\begingroup$

A laser goes wherever it is pointed, and pointing it vertically in one frame is pointing it at a slight angle in the other. You might find that easier to understand if you take an example of a person climbing up and down a vertical cliff.

Suppose the person climbs at a meter per second, and you are at the x=0 origin of your frame of reference. If you stand at the foot of the cliff while the person climbs vertically up it, then the person's y coordinate increases by 1 for every second they are climbing up, and decreases by one for every second they are climbing down, but their x coordinate doesn't change, so they are moving orthogonally to your x axis. Now suppose the person repeats the climb while you (the coordinate origin) are walking at a meter per second along the bottom of the cliff. Now the person's y coordinate changes by 1 every second, but so does their x coordinate, so relative to your walking frame they trace out a path at 45 degrees to your x-axis, and their up and down climb is a triangular path.

In order for the person to move vertically in your walking frame, they would have to follow a 45 degree path in the frame of the cliff.

The direction of all motion is therefore frame dependent. Shining a light so that its motion is vertical in one frame, means that it's not vertical in another, and vice versa.

In the thought experiments, the light is shone so it moves vertically in one frame and follows an angled path in the other. It doesn't matter which way round you do it.

You might find it interesting to consider a version of the thought experiment in which a person moving in the x direction shines two lights, one to follow a vertical up-and-down path in their frame (which is a triangular path in yours) and the other to follow a triangular path in their frame which is straight up-and-down in yours. You can now see how time dilation is reciprocal. The time the person measures for the return of the up-and-down light in their frame has to be less than the time you measure for it (so their measurement is time dilated relative to you), whereas the time they measure for the return of the angled light in their frame has to be more than the time you measure for it, so your measurement of that is time dilated in their frame.

$\endgroup$
4
$\begingroup$

Pretty much all the textbooks and videos describe this in a same way

That is very unfortunate, but it mostly just shows that you have not considered enough books / videos.

This is not a glib statement. I think all introductions of the Special Theory of Relativity (SR) that follow Einstein's original derivation should just stop. Instead, if you consult a textbook or video that starts from Minkowski diagrams first, then you will never have this confusion in the first place.

why would[n't] the light beam follow the $y$-axis?

Really, what is happening is that the man on the train has aligned his laser and mirror and detector in such a way that, for the man in the train, the light beam follows his $y$-axis. Everything should be fine here.

We already know, from the train frame of reference, that the light successfully bounces at the mirror and ends up at the detector. Everything is aligned for the train's frame of reference, everything works just fine.

But "things working just fine" is a physical statement. This means that the fact that the light reaches the detector is a physical statement that everybody has to agree upon.

So, the alignment of the $y$-axis on the train is slightly off from the $y$-axis on the ground. The laser is slanted a little bit along the $x$-axis for the ground reference frame.

That is all there is to it.

Also, when Einstein first introduced SR, there was not yet lasers, even though his annus mirabilis papers laid the foundations for them. He was talking about lamps, which are not as directionally tight as lasers are. The lamp light just goes everywhere.

$\endgroup$
2
  • $\begingroup$ Could you help me understand this: "Instead, if you consult a textbook or video that starts from Minkowski diagrams first, then you will never have this confusion in the first place." Aren't Minkowsi diagrams based on the Lorenze transformation equations? And can't those be derived from Einstein's two postulates of Special Relativity? Why would starting from Minkowski diagrams be better than starting from the two postulates of Special Relativity? $\endgroup$ Commented yesterday
  • 3
    $\begingroup$ @SyntaxJunkie Minkowski diagrams start from the same 2 postulates of SR but derives everything in a totally different way. This is very important educationally because Einstein's original argument is full of special considerations that are very easy to misunderstand, whereas Minkowski diagrams start by constructing synchronisation protocols that everybody agrees upon. Then there is no special behaviour of light clocks, because what clock type it is, is totally irrelevant. And then it can derive time dilation, length contraction, and finally piece together Lorentz transformations. $\endgroup$ Commented yesterday
4
$\begingroup$

the light beam too "lands" at the very same point it was shot from. My question is, why?

It doesn’t have to land there. The man could aim it at another location. Or the mirror could be angled at a different angle. If the man didn’t aim correctly and if the mirror wasn’t angled correctly then the light would not land at the same point.

So in the end, it lands there because we chose to have it land there. We chose to have it land there because it makes the rest of the analysis easier. We could have made a different choice, but that just would have unnecessarily complicated a difficult concept.

Why does it have to follow the movement of the platform?

If it didn’t then it would miss the mirror. And we already established that we have chosen to have the light beam hit the mirror. The two facts go together.

Now, this is possible because the light beam sort of moves with the platform.

I think you have this a bit backwards. Instead of saying that it is possible for the light to hit the mirror because it sort of moves with the platform, I would say it is necessary for the light to move with the platform because we chose to have the light hit the mirror.

$\endgroup$
1
$\begingroup$

It may help you to consider that the "pure time" direction in one frame is a mixture of time and space in a different frame. The Lorentz transform can be written:

$$cT = \gamma ~ct - \gamma \beta ~x $$

$$X = \gamma ~x - \gamma \beta ~ct $$

or if c=1:

$$T = \gamma ~t - \gamma \beta ~x $$

$$X = \gamma ~x - \gamma \beta ~t $$

You can see that each "time" direction in one frame contains components of "time" and "space" in a different frame. Just like if you have two coordinate axes $(x,y)$ and $(x',y')$, one rotated with respect to another, the pure $x$ direction will be a mixture of the $x'$ and $y'$ directions, which can be shown via a right triangle and the Pythagorean theorem.

All this to say, I think any (linear) coordinate transformation will involve right triangles and each coordinate being a linear combination of components in the other frame.

Note: $\beta = v/c$, $\gamma = 1/\sqrt{(1-\beta^2)}$

$\endgroup$
1
$\begingroup$

Why would the light beam go upwards and also "follow" the platform?

The man has aimed the laser so that the beam returns to him, and all observers must agree that the beam returns to him. Changing reference frames just changes coordinates. It doesn't change what actually occurs.

Let's say that the returning beam hits a particular spot on his hand. All observers must agree that the light hits that spot, regardless of what time and space coordinates they assign to the events of the light pulse leaving the laser, reflecting off the mirror, and hitting his hand.

Now, this is possible because the light beam sort of moves with the platform.
[...]
If it were a ball being thrown upwards, it would make sense since that ball already has the momentum of the boat, so it will cover a track that looks like a triangle from our perspective. Why would the light beam also do that?

The light beam does that because the laser inherits the horizontal momentum of the platform, and so do the photons it emits. A photon has no mass, but it does have momentum, and it behaves in accordance with the law of conservation of momentum, just like everything else.

$\endgroup$
-4
$\begingroup$

"[T]he light beam sort of moves with the platform. .... My question is, why?"

Short answer: If you're looking for a mechanism (why light behaves the way it does), nobody knows.

Longer answer: In Einstein's original formulation of special relativity, this is a presumption:

...as has already been shown to the first order of small quantities, the same laws of electrodynamics and optics will be valid for all frames of reference for which the equations of mechanics hold good. We will raise this conjecture (the purport of which will hereafter be called the “Principle of Relativity”) to the status of a postulate....

This is often phrased as "There is no preferred reference frame" or "There is no experiment that can be performed to determine whether one is at rest or moving with constant velocity."

why wouldn't the light beam follow the y-axis?

Because this would prefer the reference frame of the observer outside of the moving platform, which would violate Einstein's first postulate of Special Relativity as quoted above.

Why does it [the light beam] have to follow the movement of the platform?

If light did, somehow, take on a trajectory that was independent of the motion of the moving platform, then it would miss the mirror. This would allow the person moving with the platform to determine that his or her frame of reference was moving. And that would again contradict Einstein's first postulate.

So your question boils down to "Why is Einstein's first postulate of Special Relativity true?" And until that postulate is derived from more fundamental principles (or disproved), the only honest answer is "We don't know."

$\endgroup$

Your Answer

By clicking “Post Your Answer”, you agree to our terms of service and acknowledge you have read our privacy policy.

Start asking to get answers

Find the answer to your question by asking.

Ask question

Explore related questions

See similar questions with these tags.